Difference between revisions of "Aufgaben:Exercise 5.3Z: Non-Recursive Filter"

From LNTwww
m (Text replacement - "„" to """)
 
(4 intermediate revisions by 2 users not shown)
Line 1: Line 1:
  
{{quiz-Header|Buchseite=Stochastische Signaltheorie/Digitale Filter
+
{{quiz-Header|Buchseite=Theory_of_Stochastic_Signals/Digital_Filters
 
}}
 
}}
  
[[File:P_ID608__Sto_Z_5_3.png|right|frame|Nichtrekursives Filter]]
+
[[File:P_ID608__Sto_Z_5_3.png|right|frame|Non-recursive filter]]
Betrachtet wird das nebenstehende nichtrekursive Filter mit den Filterkoeffizienten
+
Consider the adjacent non-recursive filter with the filter coefficients
 
:$$a_0  =  1,\quad a_1  = 2,\quad a_2  =  1.$$
 
:$$a_0  =  1,\quad a_1  = 2,\quad a_2  =  1.$$
  
Gesucht sind die jeweiligen Ausgangsfolgen  $\left\langle \hspace{0.05cm}{y_\nu  } \hspace{0.05cm}\right\rangle$, wenn am Eingang folgende Wertefolgen angelegt werden:
+
We are looking for the respective output sequences  $\left\langle \hspace{0.05cm}{y_\nu  } \hspace{0.05cm}\right\rangle$ when the following value sequences are applied to the input:
  
*die "Gleichfolge"
+
*the  "DC sequence":
:$$\left\langle \hspace{0.05cm}{x_\nu  } \hspace{0.05cm}\right\rangle  = \left\langle \hspace{0.05cm}{g_\nu  } \right\rangle  = \left\langle {\;1,\;1,\;1,\;1,\;1,\;1,\;1,\;1,\;\text{...} } \hspace{0.05cm} \right\rangle .$$
+
:$$\left\langle \hspace{0.05cm}{x_\nu  } \hspace{0.05cm}\right\rangle  = \left\langle \hspace{0.05cm}{g_\nu  } \right\rangle  = \left\langle {\;1,\;1,\;1,\;1,\;1,\;1,\;1,\;1,\;\text{...} } \hspace{0.05cm} \right\rangle ,$$
  
*die "Sinusfolge" mit der Periodendauer  $T_0 = 4 \cdot T_{\rm A}$:
+
*the  "sinusoidal sequence"  with period  $T_0 = 4 \cdot T_{\rm A}$:
 
:$$\left\langle \hspace{0.05cm}{x_\nu  } \hspace{0.05cm}\right\rangle  = \left\langle \hspace{0.05cm}{s_\nu  } \right\rangle  = \left\langle {\;0,\;1,\;0, - 1,\;0,\;1,\;0, - 1,\;\text{...} } \hspace{0.05cm}\right\rangle .$$
 
:$$\left\langle \hspace{0.05cm}{x_\nu  } \hspace{0.05cm}\right\rangle  = \left\langle \hspace{0.05cm}{s_\nu  } \right\rangle  = \left\langle {\;0,\;1,\;0, - 1,\;0,\;1,\;0, - 1,\;\text{...} } \hspace{0.05cm}\right\rangle .$$
  
Line 19: Line 19:
  
  
 
+
Notes:  
 
+
*The exercise belongs to the chapter  [[Theory_of_Stochastic_Signals/Digital_Filters|Digital Filters]] in this book.
 
+
*Reference is also made to some chapters in the book  [[Signal Representation]].
''Hinweise:''
+
*The HTML5/JavaScript applet  [[Applets:Digital_Filters|"Digital Filters"]]  illustrates the subject matter of this chapter.
*Die Aufgabe gehört zum  Kapitel  [[Theory_of_Stochastic_Signals/Digitale_Filter|Digitale Filter]] im vorliegenden Buch.
 
*Bezug genommen wird auch auf einige  Kapitel im Buch  [[Signaldarstellung]].
 
 
   
 
   
  
  
  
===Fragebogen===
+
===Questions===
  
 
<quiz display=simple>
 
<quiz display=simple>
{Wie lautet die Filter&ndash;Impulsantwort&nbsp; $h(t)$?&nbsp; Zu welchem Zeitpunkt&nbsp; $\nu \cdot T_{\rm A}$&nbsp; hat die Impulsantwort ihr Maximum?
+
{What is the filter impulse response&nbsp; $h(t)$?&nbsp; At what time&nbsp; $\nu \cdot T_{\rm A}$&nbsp; does the impulse response have its maximum?
 
|type="{}"}
 
|type="{}"}
 
$\nu \ = \ $  { 1 }
 
$\nu \ = \ $  { 1 }
  
  
{Berechnen Sie den Frequenzgang&nbsp; $H(f)$.&nbsp; Wie groß ist der Wert bei $f = 0$?
+
{Calculate the frequency response&nbsp; $H(f)$.&nbsp; What is the value at $f = 0$?
 
|type="{}"}
 
|type="{}"}
 
$H(f = 0) \ =  \ $ { 4 3% }
 
$H(f = 0) \ =  \ $ { 4 3% }
  
  
{Welche Ausgangsfolge&nbsp; $\left\langle \hspace{0.05cm} {y_\nu  } \hspace{0.05cm} \right\rangle$&nbsp; ergibt sich für die Gleichfolge&nbsp; $\left\langle \hspace{0.05cm} {g_\nu  } \hspace{0.05cm}\right\rangle$&nbsp; an seinem Eingang?&nbsp; Interpretieren Sie dieses Ergebnis unter Berücksichtigung der letzten Teilaufgabe. <br>Welcher Ausgangswert ergibt sich für&nbsp; $\nu = 4 $?
+
{What is the output sequence&nbsp; $\left\langle \hspace{0.05cm} {y_\nu  } \hspace{0.05cm} \right\rangle$&nbsp; for the&nbsp; "DC sequence" &nbsp; $\left\langle \hspace{0.05cm} {g_\nu  } \hspace{0.05cm}\right\rangle$&nbsp; at its input?&nbsp; Interpret this result considering the last subtask. <br>What is the output value for&nbsp; $\nu = 4 $?
 
|type="{}"}
 
|type="{}"}
 
$y_4 \ = \ $  { 4 3% }
 
$y_4 \ = \ $  { 4 3% }
  
  
{Welche Ausgangsfolge&nbsp; $\left\langle \hspace{0.05cm}{y_\nu  } \hspace{0.05cm}\right\rangle$&nbsp; ergibt sich für die Sinusfolge $\left\langle \hspace{0.05cm}{s_\nu  } \hspace{0.05cm}\right\rangle$ am Eingang?&nbsp; Welcher Ausgangswert ergibt sich für&nbsp; $\nu = 4 $?
+
{What output sequence&nbsp; $\left\langle \hspace{0.05cm}{y_\nu  } \hspace{0.05cm}\right\rangle$&nbsp; results for the&nbsp; "sinusoidal sequence" &nbsp; $\left\langle \hspace{0.05cm}{s_\nu  } \hspace{0.05cm}\right\rangle$ at its input?&nbsp; <br>What output value results for&nbsp; $\nu = 4 $?
 
|type="{}"}
 
|type="{}"}
 
$y_4 \ = \ $ { -2.06--1.94 }
 
$y_4 \ = \ $ { -2.06--1.94 }
Line 54: Line 52:
 
</quiz>
 
</quiz>
  
===Musterlösung===
+
===Solution===
 
{{ML-Kopf}}
 
{{ML-Kopf}}
'''(1)''' &nbsp; Die Impulsantwort lautet: &nbsp; $h(t) = \delta (t) + 2 \cdot \delta ( {t - T_{\rm A} } ) + \delta ( {t - 2T_{\rm A} } ).$
+
'''(1)''' &nbsp; The impulse response is: &nbsp; $h(t) = \delta (t) + 2 \cdot \delta ( {t - T_{\rm A} } ) + \delta ( {t - 2T_{\rm A} } ).$
*Das Maximum liegt bei&nbsp; $T_{\rm A}$, &nbsp; &rArr; &nbsp; $\underline{\nu = 1}$.
+
*The maximum is at&nbsp; $T_{\rm A}$ &nbsp; &rArr; &nbsp; $\underline{\nu = 1}$.
  
  
  
'''(2)''' &nbsp; Der Frequenzgang&nbsp; $H(f)$&nbsp; ist die Fouriertransformierte der Impulsantwort&nbsp; $h(t)$.  
+
'''(2)''' &nbsp; The frequency response&nbsp; $H(f)$&nbsp; is the Fourier transform of the impulse response&nbsp; $h(t)$.  
*Die um&nbsp; $T_{\rm A}$&nbsp; nach links verschobene Impulsantwort
+
*The impulse response shifted to the left by&nbsp; $T_{\rm A}$,&nbsp;  
:$$h\hspace{0.05cm}'(t) = \delta ( {t + T_{\rm A} } ) + 2 \cdot \delta ( t ) + \delta ( {t - T_{\rm A} } )$$
+
::$$h\hspace{0.05cm}'(t) = \delta ( {t + T_{\rm A} } ) + 2 \cdot \delta ( t ) + \delta ( {t - T_{\rm A} } ),$$
  
:ist symmetrisch um&nbsp; $t= 0$&nbsp; und hat dementsprechend den rein reellen Frequenzgang
+
:is symmetric at&nbsp; $t= 0$&nbsp; and accordingly has the purely real frequency response
:$$H\hspace{0.05cm}'(f) = 2\big [ {1 + \cos ( {2{\rm{\pi }}fT_{\rm A} } )} \big ].$$
+
::$$H\hspace{0.05cm}'(f) = 2\big [ {1 + \cos ( {2{\rm{\pi }}fT_{\rm A} } )} \big ].$$
  
*Durch Anwendung des Verschiebungssatzes folgt weiter:
+
*By applying the shifting theorem,&nbsp; it further follows:
 
:$$H(f) = 2\big [ {1 + \cos ( {2{\rm{\pi }}fT_{\rm A} } )} \big ] \cdot {\rm{e}}^{ - {\rm{j}}2{\rm{\pi }}fT_{\rm A} } .$$
 
:$$H(f) = 2\big [ {1 + \cos ( {2{\rm{\pi }}fT_{\rm A} } )} \big ] \cdot {\rm{e}}^{ - {\rm{j}}2{\rm{\pi }}fT_{\rm A} } .$$
*Der Wert des Frequenzgangs bei der Frequenz $f=0$ ist demzufolge $H(f = 0)\hspace{0.15cm}\underline{ = 4}$.
+
*Consequently,&nbsp; the value of the frequency response at frequency&nbsp; $f=0$&nbsp; is&nbsp;
 +
:$$H(f = 0)\hspace{0.15cm}\underline{ = 4}.$$
  
  
  
'''(3)''' &nbsp; Die zeitdiskrete Faltung der Eingangsfolge&nbsp; $\left\langle \hspace{0.05cm}{g_\nu  } \hspace{0.05cm} \right\rangle$&nbsp; mit der&nbsp; Impulsantwort $\left\langle \hspace{0.05cm}{h_\nu  } \hspace{0.05cm}\right\rangle = \left\langle \hspace{0.05cm}{1, 2, 1  } \hspace{0.05cm}\right\rangle$&nbsp; ergibt
+
'''(3)''' &nbsp; The discrete-time convolution of the input sequence&nbsp; $\left\langle \hspace{0.05cm}{g_\nu  } \hspace{0.05cm} \right\rangle$&nbsp; with the impulse response&nbsp; $\left\langle \hspace{0.05cm}{h_\nu  } \hspace{0.05cm}\right\rangle = \left\langle \hspace{0.05cm}{1, 2, 1  } \hspace{0.05cm}\right\rangle$&nbsp; results in
 
:$$\left\langle \hspace{0.05cm}{y_\nu  } \hspace{0.05cm}\right\rangle  = \left\langle {\;1,\;3,\;4,\;4,\;4,\;4,\;4,\;4,\;4,\;4,\;4,\; \text{...} \;} \right\rangle $$
 
:$$\left\langle \hspace{0.05cm}{y_\nu  } \hspace{0.05cm}\right\rangle  = \left\langle {\;1,\;3,\;4,\;4,\;4,\;4,\;4,\;4,\;4,\;4,\;4,\; \text{...} \;} \right\rangle $$
  
*Insbesondere gilt&nbsp; $y_4\hspace{0.15cm}\underline{ = 4}$.  
+
*In particular,&nbsp; $y_4\hspace{0.15cm}\underline{ = 4}$.  
*Mit Ausnahme der Werte&nbsp; $y_0$&nbsp; und&nbsp; $y_1$&nbsp; (Einschwingvorgang) erhält man auch am Ausgang eine Gleichfolge mit dem konstanten Wert 4:  
+
*With the exception of the values&nbsp; $y_0$&nbsp; and&nbsp; $y_1$&nbsp; (transient), we also obtain a synchronous sequence at the output with the constant value 4:  
 
:$$y(t) = H( {f = 0} ) \cdot x( t ) = 4 \cdot 1 = 4.$$
 
:$$y(t) = H( {f = 0} ) \cdot x( t ) = 4 \cdot 1 = 4.$$
  
  
  
'''(4)''' &nbsp; Analog zur Teilaufgabe&nbsp; '''(3)'''&nbsp; erhält man nun durch Verschiebung, Gewichtung mit&nbsp; $a_1$,&nbsp; $a_2$,&nbsp; $a_3$&nbsp; und anschließender Überlagerung:
+
'''(4)''' &nbsp; Analogous to subtask&nbsp; '''(3)''',&nbsp; we now obtain by shifting, weighting with&nbsp; $a_1$,&nbsp; $a_2$,&nbsp; $a_3$&nbsp; and subsequent superposition:
 
:$$\left\langle \hspace{0.05cm}{y_\nu  } \hspace{0.05cm}\right\rangle  = \left\langle \hspace{0.05cm}{\;0,\;1,\;2,\;0,\; - 2,\;0,\;2,\;0,\; - 2,\;0,\;...\;}\hspace{0.05cm} \right\rangle .$$
 
:$$\left\langle \hspace{0.05cm}{y_\nu  } \hspace{0.05cm}\right\rangle  = \left\langle \hspace{0.05cm}{\;0,\;1,\;2,\;0,\; - 2,\;0,\;2,\;0,\; - 2,\;0,\;...\;}\hspace{0.05cm} \right\rangle .$$
  
*Der gesuchte Wert ist somit&nbsp; $y_4\hspace{0.15cm}\underline{ = -2}$.
+
*Thus,&nbsp; the value we are looking for is&nbsp; $y_4\hspace{0.15cm}\underline{ = -2}$.
  
  
<b>Anderer Lösungsweg:</b>  
+
<b>Another solution:</b>  
*Die Eingangsfolge&nbsp; $\left\langle \hspace{0.05cm}{s_\nu  }\hspace{0.05cm} \right\rangle$&nbsp; verläuft sinusförmig mit der Periode&nbsp; $4 \cdot T_{\rm A}$.&nbsp; Die Grundfrequenz ist dementsprechend&nbsp; $f_0 = 1/(4 \cdot T_{\rm A})$.  
+
*The input sequence&nbsp; $\left\langle \hspace{0.05cm}{s_\nu  }\hspace{0.05cm} \right\rangle$&nbsp; is sinusoidal with period&nbsp; $4 \cdot T_{\rm A}$.&nbsp; Accordingly, the basic frequency is&nbsp; $f_0 = 1/(4 \cdot T_{\rm A})$.  
*Bei dieser Frequenz hat der Frequenzgang&nbsp; $H(f)$&nbsp; gemäß Teilaufgabe&nbsp; '''(2)'''&nbsp; den folgenden Wert:
+
*At this frequency, the frequency response&nbsp; $H(f)$&nbsp; has the following value according to subtask&nbsp; '''(2)''':&nbsp;  
 
:$$H( {f = f_0 } ) = 2\big[ {1 + \cos ( {{{\rm{\pi }}}/{2}} )} \big] \cdot {\rm{e}}^{ - {\rm{j\pi /2}}}  = 2 \cdot {\rm{e}}^{ - {\rm{j\pi /2}}} .$$
 
:$$H( {f = f_0 } ) = 2\big[ {1 + \cos ( {{{\rm{\pi }}}/{2}} )} \big] \cdot {\rm{e}}^{ - {\rm{j\pi /2}}}  = 2 \cdot {\rm{e}}^{ - {\rm{j\pi /2}}} .$$
*Lässt man den Einschwingungsvorgang&nbsp; $($abgeschlossen bei&nbsp; $t = T_{\rm A})$&nbsp; außer Betracht, so ergibt sich mit&nbsp; $\tau = T_{\rm A}$ &nbsp; $($Phase: &nbsp; $90^\circ)$&nbsp; folgender Zusammenhang zwischen dem Eingangs- und dem Ausgangssignal:
+
*Leaving the transient&nbsp; $($completed at&nbsp; $t = T_{\rm A})$&nbsp; out of consideration,&nbsp; the following relationship between the input and the output signals is obtained with&nbsp; $\tau = T_{\rm A}$ &nbsp; $($phase: &nbsp; $90^\circ)$:&nbsp;  
 
:$$y(t) = 2 \cdot x( {t - T_{\rm A} } ).$$
 
:$$y(t) = 2 \cdot x( {t - T_{\rm A} } ).$$
*Das heißt: &nbsp; Aus der Sinusfunktion wird die <u>Funktion "Minus-Cosinus" mit der Amplitude 2</u>.
+
*That means: &nbsp; <u>The sine function becomes the&nbsp; function "minus-cosine"&nbsp; with the amplitude 2</u>.
  
 
{{ML-Fuß}}
 
{{ML-Fuß}}
Line 101: Line 100:
  
  
[[Category:Theory of Stochastic Signals: Exercises|^5.2 Digitale Filter^]]
+
[[Category:Theory of Stochastic Signals: Exercises|^5.2 Digital Filters^]]

Latest revision as of 19:16, 10 February 2022

Non-recursive filter

Consider the adjacent non-recursive filter with the filter coefficients

$$a_0 = 1,\quad a_1 = 2,\quad a_2 = 1.$$

We are looking for the respective output sequences  $\left\langle \hspace{0.05cm}{y_\nu } \hspace{0.05cm}\right\rangle$ when the following value sequences are applied to the input:

  • the  "DC sequence":
$$\left\langle \hspace{0.05cm}{x_\nu } \hspace{0.05cm}\right\rangle = \left\langle \hspace{0.05cm}{g_\nu } \right\rangle = \left\langle {\;1,\;1,\;1,\;1,\;1,\;1,\;1,\;1,\;\text{...} } \hspace{0.05cm} \right\rangle ,$$
  • the  "sinusoidal sequence"  with period  $T_0 = 4 \cdot T_{\rm A}$:
$$\left\langle \hspace{0.05cm}{x_\nu } \hspace{0.05cm}\right\rangle = \left\langle \hspace{0.05cm}{s_\nu } \right\rangle = \left\langle {\;0,\;1,\;0, - 1,\;0,\;1,\;0, - 1,\;\text{...} } \hspace{0.05cm}\right\rangle .$$



Notes:



Questions

1

What is the filter impulse response  $h(t)$?  At what time  $\nu \cdot T_{\rm A}$  does the impulse response have its maximum?

$\nu \ = \ $

2

Calculate the frequency response  $H(f)$.  What is the value at $f = 0$?

$H(f = 0) \ = \ $

3

What is the output sequence  $\left\langle \hspace{0.05cm} {y_\nu } \hspace{0.05cm} \right\rangle$  for the  "DC sequence"   $\left\langle \hspace{0.05cm} {g_\nu } \hspace{0.05cm}\right\rangle$  at its input?  Interpret this result considering the last subtask.
What is the output value for  $\nu = 4 $?

$y_4 \ = \ $

4

What output sequence  $\left\langle \hspace{0.05cm}{y_\nu } \hspace{0.05cm}\right\rangle$  results for the  "sinusoidal sequence"   $\left\langle \hspace{0.05cm}{s_\nu } \hspace{0.05cm}\right\rangle$ at its input? 
What output value results for  $\nu = 4 $?

$y_4 \ = \ $


Solution

(1)   The impulse response is:   $h(t) = \delta (t) + 2 \cdot \delta ( {t - T_{\rm A} } ) + \delta ( {t - 2T_{\rm A} } ).$

  • The maximum is at  $T_{\rm A}$   ⇒   $\underline{\nu = 1}$.


(2)   The frequency response  $H(f)$  is the Fourier transform of the impulse response  $h(t)$.

  • The impulse response shifted to the left by  $T_{\rm A}$, 
$$h\hspace{0.05cm}'(t) = \delta ( {t + T_{\rm A} } ) + 2 \cdot \delta ( t ) + \delta ( {t - T_{\rm A} } ),$$
is symmetric at  $t= 0$  and accordingly has the purely real frequency response
$$H\hspace{0.05cm}'(f) = 2\big [ {1 + \cos ( {2{\rm{\pi }}fT_{\rm A} } )} \big ].$$
  • By applying the shifting theorem,  it further follows:
$$H(f) = 2\big [ {1 + \cos ( {2{\rm{\pi }}fT_{\rm A} } )} \big ] \cdot {\rm{e}}^{ - {\rm{j}}2{\rm{\pi }}fT_{\rm A} } .$$
  • Consequently,  the value of the frequency response at frequency  $f=0$  is 
$$H(f = 0)\hspace{0.15cm}\underline{ = 4}.$$


(3)   The discrete-time convolution of the input sequence  $\left\langle \hspace{0.05cm}{g_\nu } \hspace{0.05cm} \right\rangle$  with the impulse response  $\left\langle \hspace{0.05cm}{h_\nu } \hspace{0.05cm}\right\rangle = \left\langle \hspace{0.05cm}{1, 2, 1 } \hspace{0.05cm}\right\rangle$  results in

$$\left\langle \hspace{0.05cm}{y_\nu } \hspace{0.05cm}\right\rangle = \left\langle {\;1,\;3,\;4,\;4,\;4,\;4,\;4,\;4,\;4,\;4,\;4,\; \text{...} \;} \right\rangle $$
  • In particular,  $y_4\hspace{0.15cm}\underline{ = 4}$.
  • With the exception of the values  $y_0$  and  $y_1$  (transient), we also obtain a synchronous sequence at the output with the constant value 4:
$$y(t) = H( {f = 0} ) \cdot x( t ) = 4 \cdot 1 = 4.$$


(4)   Analogous to subtask  (3),  we now obtain by shifting, weighting with  $a_1$,  $a_2$,  $a_3$  and subsequent superposition:

$$\left\langle \hspace{0.05cm}{y_\nu } \hspace{0.05cm}\right\rangle = \left\langle \hspace{0.05cm}{\;0,\;1,\;2,\;0,\; - 2,\;0,\;2,\;0,\; - 2,\;0,\;...\;}\hspace{0.05cm} \right\rangle .$$
  • Thus,  the value we are looking for is  $y_4\hspace{0.15cm}\underline{ = -2}$.


Another solution:

  • The input sequence  $\left\langle \hspace{0.05cm}{s_\nu }\hspace{0.05cm} \right\rangle$  is sinusoidal with period  $4 \cdot T_{\rm A}$.  Accordingly, the basic frequency is  $f_0 = 1/(4 \cdot T_{\rm A})$.
  • At this frequency, the frequency response  $H(f)$  has the following value according to subtask  (2)
$$H( {f = f_0 } ) = 2\big[ {1 + \cos ( {{{\rm{\pi }}}/{2}} )} \big] \cdot {\rm{e}}^{ - {\rm{j\pi /2}}} = 2 \cdot {\rm{e}}^{ - {\rm{j\pi /2}}} .$$
  • Leaving the transient  $($completed at  $t = T_{\rm A})$  out of consideration,  the following relationship between the input and the output signals is obtained with  $\tau = T_{\rm A}$   $($phase:   $90^\circ)$: 
$$y(t) = 2 \cdot x( {t - T_{\rm A} } ).$$
  • That means:   The sine function becomes the  function "minus-cosine"  with the amplitude 2.